LSAT and Law School Admissions Forum

Get expert LSAT preparation and law school admissions advice from PowerScore Test Preparation.

 Administrator
PowerScore Staff
  • PowerScore Staff
  • Posts: 8916
  • Joined: Feb 02, 2011
|
#84978
Complete Question Explanation

The correct answer choice is (E).

Answer choice (A):

Answer choice (B):

Answer choice (C):

Answer choice (D):

Answer choice (E): This is the correct answer choice.

This explanation is still in progress. Please post any questions below!
User avatar
 slynnnnn
  • Posts: 9
  • Joined: Jan 03, 2022
|
#92993
Hi PowerScore,

I did think (E) to be the correct answer until I saw "it is based on an assumption that cannot be verified using the archaeological record."

To me, the flaw pointed by the author is not about whether the archaeological record can verify John Lowe's assumption. Instead, it's about whether the archaeological record itself is a verifiable source for the assumption to be built on. Upon that consideration, I eliminate (E).

Can you please help me to explain that? Thanks!

-Lynn
 Robert Carroll
PowerScore Staff
  • PowerScore Staff
  • Posts: 1787
  • Joined: Dec 06, 2013
|
#93022
slynnnnn,

It seems like you've justified the language in answer choice (E) after all! If the archaeological record is not a verifiable source for the assumption to be built on, then surely the assumption can't be verified that way! Otherwise, that would be circular reasoning.

Further, you don't indicate what answer choice you picked instead. As I just showed, your reasoning actually supports answer choice (E), but even if we had doubts about it, what answer would you switch to? The first three answer choices are all uncritically praising of Lowe, and it's central to the passage that Lowe is at least somewhat mistaken, so those answers can't be picked. Answer choice (D) has a huge problem - it requires us to know the circumstances of the collapse of the Mayans, so that we can see that Lowe is wrong about what we already know. But the author's point is that it's really difficult to know what actually happened with the Mayans, so it's difficult to prove Lowe right...but then again, it's also difficult to prove him wrong. Although the first paragraph of my post here shows that we can have no doubts about answer choice (E) anyway, even if we did, you'd have to compare them to the much more serious doubts about all the other answers in this case.

Robert Carroll
User avatar
 slynnnnn
  • Posts: 9
  • Joined: Jan 03, 2022
|
#93024
Robert Carroll wrote: Tue Jan 04, 2022 1:16 pm slynnnnn,

It seems like you've justified the language in answer choice (E) after all! If the archaeological record is not a verifiable source for the assumption to be built on, then surely the assumption can't be verified that way! Otherwise, that would be circular reasoning.

Further, you don't indicate what answer choice you picked instead. As I just showed, your reasoning actually supports answer choice (E), but even if we had doubts about it, what answer would you switch to? The first three answer choices are all uncritically praising of Lowe, and it's central to the passage that Lowe is at least somewhat mistaken, so those answers can't be picked. Answer choice (D) has a huge problem - it requires us to know the circumstances of the collapse of the Mayans, so that we can see that Lowe is wrong about what we already know. But the author's point is that it's really difficult to know what actually happened with the Mayans, so it's difficult to prove Lowe right...but then again, it's also difficult to prove him wrong. Although the first paragraph of my post here shows that we can have no doubts about answer choice (E) anyway, even if we did, you'd have to compare them to the much more serious doubts about all the other answers in this case.

Robert Carroll
Hi Robert,

Thank you for your explanation. I do get your point now!

When doing the practice test, I actually ended up eliminating all of the answer choices. I understood why (a)-(d) were all wrong. I guess I was just getting myself into a dead end with emphasizing too much on this specific statement, where I thoroughly believed that the assumption could in fact be verified by the archaeological record, and that this was a huge problem as serious as all the other answer choices.

Thanks again!
Lynn

Get the most out of your LSAT Prep Plus subscription.

Analyze and track your performance with our Testing and Analytics Package.